LSAT and Law School Admissions Forum

Get expert LSAT preparation and law school admissions advice from PowerScore Test Preparation.

 Administrator
PowerScore Staff
  • PowerScore Staff
  • Posts: 8915
  • Joined: Feb 02, 2011
|
#22657
Question #17: Assumption. The correct answer choice is (B)

The argument in #17 appears in the last sentence, where the author concludes that for rockets to work most effectively throughout their ascents, all rocket engines must have both short nozzles and long nozzles. Note the strong language in that statement—“all” and “must” are both absolute terms.

How does the author attempt to justify such a strong claim? We’re told that rocket engines are most effective when the pressure of escaping exhaust gases matches that of the surrounding atmosphere. But this presents a conundrum: at low altitudes atmospheric pressure is high and a short nozzle is most effective, whereas the pressure at higher altitudes is presumably lower (“thin upper atmosphere”) and a long nozzle becomes more effective.

And that explains why the author believe both nozzle lengths are necessary.

But think about what the author is presuming with that conclusion: all rockets will pass through both the lower and upper atmosphere! If they don’t—maybe they’re strictly low-altitude rockets, or maybe they’re being fired from a craft that’s already in the upper atmosphere—then having both nozzle types doesn’t make any sense.

And this is a good exercise for Assumption questions, particularly Defender Assumptions like this one: if you can spot what would make the conclusion seem unreasonable, like a lower-atmosphere-only rocket in this case, then you know the author is assuming that idea doesn’t happen or exist. In other words all rockets leave the lower atmosphere and pass through the upper.

Answer choice (A): Difficulty of construction or assembly doesn’t matter in this argument; the conclusion is strictly about what makes all rockets work most effectively.

Answer choice (B): This is the correct answer choice. As noted above, the author must assume that all rockets will pass through the upper atmosphere at some point, otherwise there is no reason to suggest that all rockets are improved by the addition of long nozzles.

If you apply the Assumption Negation Technique to this you see very quickly the damage the negated version would do: not all rockets pass through the upper atmosphere. Hence, not all rockets would need long nozzles, and the author would be incorrect. That test is a powerful tool against this question type.

Answer choice (C): The conclusion is about effectiveness, not absolute capability. How high short nozzles can take a rocket is irrelevant.

Answer choice (D): Be careful. This reads at first like a pure restatement (which is not the same as an assumption, I should point out), but it gets a crucial idea wrong: the stimulus describes how to make a rocket engine work “most effectively,” whereas (D) goes with the less qualified “to work effectively” (no “most”). Discussing general effectiveness is not the same thing as striving for the maximum, so not only is (D) not an assumption, it is an outright mischaracterization of the information provided.

Answer choice (E): Again, be careful. The conclusion is that all rockets must have short and long nozzles on “their engines,” meaning all engines. So reading about “at least one” misstates what we’ve been told. Secondly, providing a less absolute version of the conclusion does not provide an assumption of the author—we know what this author believes, so the key is considering what unspoken information is central to that belief. Be wary of seeming restatements (as we saw in the previous answer).
 TOgren2424
  • Posts: 17
  • Joined: May 21, 2017
|
#35790
Still having a little bit of trouble with seeing how D is not an assumption

Conclusion: Thus, to work most effectively throughout their ascents, all rockets must have both short nozzle and long nozzles on their engines.

Premise: Rocket engines are most effective when exhaust gases escape from their nozzles at the same pressure as the surrounding atmosphere. Low altitudes= short nozzle, high altitudes= long nozzle.

Is it not an assumption of the argument that for a rocket to be most effective, its engines must be most effective. Although (D) states for a rocket to "work effectively" and not "most effectively," this is still a necessary assumption, right? In order to be the work most effectively, you have to work effectively.
 Francis O'Rourke
PowerScore Staff
  • PowerScore Staff
  • Posts: 471
  • Joined: Mar 10, 2017
|
#35796
I see what you are saying here, but unfortunately, it does not work out the way you are describing. Rocket exhaust gas escape pressure is a bit tricky to talk about, so I want to use a parallel situation. Imagine that I said: 'in order for our team to play most effectively tonight, we must complete each pass exactly as we practiced.' In this case could we play effectively and still make some very slight errors in our play? Your thinking and answer choice (D) would say no: if we do anything differently, we cannot play effectively at all. In reality, we have no information about what is required to play effectively; we only information about what is required to play most effectively.

Here's another situation: 'This plant will grow tallest when it receives exactly 0.5 liters of water per day.' Could this plant grow tall if it only receives 0.45 Liters of water per day? Perhaps, since the rule I just provided does not tell us what is required for a plant to grow tall, rather it only tells us what is required for it to grow to its tallest height.

The mistake that you are making is assuming that if something is required for the most X then it is required for any X. The relationship is actually the reverse of what you said: if something is required for the X, then it is required for having the most X. I hope that you can see through those two examples above that this is a mistake.
 nlittle
  • Posts: 18
  • Joined: Sep 09, 2017
|
#40431
So just to clarify on answer choice E, the answer was only wrong because of the second point made about restatements?

In the first point, the use of all seems to actually imply that least one has both nozzles if all do. Also, just to confirm on the second point, does this mean that an answer choice is required to contain unspoken information in order to be an assumption?

Further, with regard to answer choice D discussed above, does using 'most effective' mean that something can be the most effective that it can be without being considered 'effective'? As in the plant example, you can give the plant the optimal amount of water to maximize its height, however, the species of plant itself could be rather short and therefore the plant may not be considered tall regardless? Please let me know if this is analogous to your point.

Thanks,

Nick
 James Finch
PowerScore Staff
  • PowerScore Staff
  • Posts: 943
  • Joined: Sep 06, 2017
|
#40487
Hi Nick,

An assumption is an unstated premise, and the correct answer to an assumption question will fill in a gap between the stated premises and the conclusion by either directly linking the premises to a new element in the conclusion (a Supporter assumption) or by fending off a possible line of attack on the conclusion (Defender assumption). In this case, (B) acts as a Supporter assumption, linking the conclusion's inclusion of both nozzles being necessary to the earlier premises about how a rocket engine operates at maximum effectiveness.

Answer choice (E) is essentially a separate conclusion entirely supported by the stated premises in the stimulus, not requiring an assumption to be valid.

With (D), the issue you raised is correct, a rocket could work effectively without being at its most effective, just like a person or plant could be tall without being as tall as possible or the tallest, or as you state, a plant could reach its tallest possible height without being tall generally.

Hope this helps!
 jmramon
  • Posts: 47
  • Joined: Jul 21, 2017
|
#41522
Hi,

So I’m confused with the difference between supporter and defender answers with assumption questions after reading this thread. Unless I’m mistaken, I thought the first post by the Administrator said B was a defender answer, whereas James says it’s a supporter answer. I guess B could be seen as a defender because it eliminates a significant critique of the stimulus’ conclusion (so the argument depends on B being the case, as we look for in assumption answers), but then again, as James states, it could be seen as a support premise to add to the conclusion by filling the unstated gap between the initial premises and conclusion. Could someone please state the difference between a supporter and defender answer? I thought I understood it clearly before, but now if a question can be seen as both I’m doubting my initial confidence in knowing the difference. Thank you :-D
 Claire Horan
PowerScore Staff
  • PowerScore Staff
  • Posts: 408
  • Joined: Apr 18, 2016
|
#42127
Hi jmramon,

Your explanation was spot on, so I don't think you are at all confused about supporter and defender assumptions. An assumption is an assumption.

Here's how I approach assumption questions:

While reading a stimulus, I note whether there are any obvious gaps in the argument. If I see one, I try to prephrase the assumption being made. I can often prephrase these supporter assumptions and just look for the answer choice that matches.

I would say the weaker the argument, the more likely you will be able to prephrase a supporter assumption. Defender assumptions, on the other hand, or more likely when an argument is relatively strong. Since there are no obvious gaps, you have to think of an argument against the stimulus argument (a chink in the armor), and then find a way to eliminate that weakness.

So the concepts of "supporter" and "defender" assumptions help to describe different ways to look for assumptions.
 ltowns1
  • Posts: 61
  • Joined: May 16, 2017
|
#45498
Could you make a case that (E) was wrong because of the first part of the conclusion as well? The stimulus says "works most effective throughout their ascent",whereas (E) says "at both low and and high atmospheric pressure"? Or is that more of a distinction without a difference? The Engines detail snuck right pass me.
 Daniel Stern
PowerScore Staff
  • PowerScore Staff
  • Posts: 81
  • Joined: Feb 07, 2018
|
#45506
See the original administrator post on this thread for an excellent discussion of incorrect answer choice E:

E is a watered down restatement of the argument's conclusion, as opposed to an assumption necessary to reach the conclusion.

You point out that E has swapped out "throughout the ascent" from the stimulus conclusion for "at both low and high" altitudes," and this in fact turns out to be the critical link that necessary assumption and credited answer choice A provides for us. It is the "ascent's" inclusion of travel through both portions of the atmosphere that makes the conclusion follow: if some rockets are flying just in the lower atmosphere without ever reaching the higher atmosphere, then those rockets would not need both kinds of nozzles on their engines.

So although E has similar language to what we want, including both kinds of atmosphere, E then goes astray by talking about "at least one" engine has to have the nozzles, while our conclusion that we want to support is that all the engines have to have both kinds of nozzles.

I have edited this post to clarify that i am discussing incorrect answer E which the prior post asked about: the correct answer here is B, which posits that the "ascent" includes both types of atmosphere, thus necessitating both types of nozzles on the rocket engines.

Good luck in your studies,
Dan
 ltowns1
  • Posts: 61
  • Joined: May 16, 2017
|
#45510
I think you mean (B) as the credited answer. (Just wanted to clarify for anyone reading this in the future) So just to be clear, it does mean the same thing and thus it would not be a reason to get rid of this answer choice?


The discussion up top did help a lot. Especially when you think about the fact that (E) just gives us a rule that really isn't all that useful in the first place. My initial thoughts made me question if we knew whether the rockets do indeed go through both atmospheres. (High and low) B tells us that,while (E) is really just a rule..I think.

Get the most out of your LSAT Prep Plus subscription.

Analyze and track your performance with our Testing and Analytics Package.